Which expression is the simplest form of - (4x³ + x²) + 2 (x³ - 3x²)?
OA.-223-5x²
B. -6x³6x²
OC.-6x³2x²
OD. -2x³-7x²
SUBMIT

Answers

Answer 1
Distribute what is outside the parenthesis first (-&2) to get

-4x^3-x^2+2x^3-6x^2

Then combine like terms to get

-2x^3-7x^2

So your answer is D.


Hope this helped :)
Answer 2

Answer:

-2x³ - 7x².

Step-by-step explanation:

- (4x³ + x²) + 2(x³ - 3x²)

= - 4x³ - x² + 2x³ - 6x²    Simplifying like terms:

= -2x³ - 7x².


Related Questions

Why might the t-test be more useful in actual practice when you are running statistics rather than a z-test? give an example of a situation where you might use each of these and label their pros and cons.

Answers

The t-test might be more useful in actual practice when standard deviation or variance is unknown.

What is a t-test?

The t-test is used to determine how the averages of various data sets differ from one another.

When variance is not provided, the T-test, a particular kind of parametric test, is used to determine how the means of two sets of data differ from one another. When variance is provided, the Z-test indicates a hypothesis test that determines whether the means of two datasets differ from one another.

Example of t-test: If you flip a coin 1,000 times, for instance, you can discover that the outcome is distributed normally over all trials.

Example of z-test: We are conducting research using information gathered from cohorts of students who have taken Elementary Statistics in the past.

T-tests are typically more suitable when addressing issues with small sample sizes, whereas z-tests are suitable for large sample sizes.

Using the t-test for an ordinal variable, their frequency is not even close to a normal distribution, and the arithmetic mean offers an unsuitable measure of location.

Because we frequently don't know the population standard deviation, Z-Tests have this drawback.

Learn more about t-tests here:

https://brainly.com/question/6589776

#SPJ4

Step 1: –10 + 8x < 6x – 4
Step 2: –10 < –2x – 4
Step 3: –6 < –2x
Step 4: ________

What is the final step in solving the inequality –2(5 – 4x) < 6x – 4?

x < –3
x > –3
x < 3
x > 3

Answers

Answer:

x < 3

Step-by-step explanation:

The last step is to divide by -2. When you divide by a negative number you must reverse (flip) the inequality symbol.

-6 < -2x

-6/-2 > -2x/-2

3 > x

This is the same as

x < 3 (see the pointy side pointing at the x and the wide open side facing the 3)

Jose began the day with eight Magikarps. After spending the day at the lake, he ended up with sixteen Magikarps.

Find the Percent of increase
thats all she wrote

Answers

A percentage is a way to describe a part of a whole. The percentage increase of Magikarps is 100%.

What are Percentages?

A percentage is a way to describe a part of a whole. such as the fraction ¼ can be described as 0.25 which is equal to 25%.

To convert a fraction to a percentage, convert the fraction to decimal form and then multiply by 100 with the '%' symbol.

Jose began the day with eight Magikarps. After spending the day at the lake, he ended up with sixteen Magikarps. Therefore, The percentage increase of Magikarps is,

Percentage increase = (16-8)/8 × 100% = 100%

Hence, The percentage increase of Magikarps is 100%.

Learn more about Percentages:

https://brainly.com/question/6972121

#SPJ1

If the nth term of a sequence is 11n+2 what is the 12th term

Answers

Answer:

134

Step-by-step explanation:

the "nth" term of a sequence means that you can plug in the term number to get the term value

so, if

  11n + 2 is the sequence

  11(12) + 2 is the 12th term

  132 + 2 is the 12th term

  134 is the 12th term

By plugging in the term "place" (1st, 2nd, 3rd, 4th, etc.) for the variable "n", we get a term value.

hope this helps!!

The 12th term of the sequence is 133

How to determine the 12th term?

The nth term is given as:

Tn = 11n + 1

The 12th term is calculated as:

T(12) = 11(12) + 1

Evaluate

T(12) = 133

Hence, the 12th term of the sequence is 133

Read more about sequence at:

https://brainly.com/question/6561461

#SPJ6



Which series of transformations will not map figure Q onto itself?
O(x+2, y-2), reflection over y = x - 1
O(x-2, y + 2), reflection over y = x + 3
O(x+0y-4), reflection over y = x - 1
O(x-3, y-3), reflection over y = -x +1
Question 1 (Answered)
OF

Answers

Your question is incomplete. Below you will find the missing content.

Which series of transformations will not map figure H onto itself?

A. O(x+0, y-2), reflection over y = 1

B. O(x+2, y-0), reflection over x=3

C. O(x+3, y+3), reflection over y = -x +7

D. O(x-3, y-3), reflection over y = -x +2

The correct option is Option D: the series of transformation O(x-3, y-3), reflection over y = -x +2 will not map figure H onto itself.

Here given a square with its 4 vertices at coordinates (2,1), (1,2), (2,3) and (3,2).

By checking all the options

Option A:

1st transformation (x+0, y-2) will map 4 vertices of the square into points of coordinates such as

(2,1) → (2,-1)

(1,2) → (1,0)

(2,3) → (2,1)

(3,2) → (3,0)

next transformation which is the reflection over y=1 gives the coordinates (x,2-y)

(2,-1) → (2,3)

(1,0) → (1,2)

(2,1) → (2,1)

(3,0) → (3,2)

These new points are exactly the same as the vertices of the initial square.

Option B:

1st transformation (x+2, y-0) will map 4 vertices of the square into points of coordinates such as

(2,1) → (4,1)

(1,2) → (3,2)

(2,3) → (4,3)

(3,2) → (5,2)

next transformation which is the reflection over x=3 gives the coordinates (6-x,y)

(4,1) → (2,1)

(3,2) → (3,2)

(4,3) → (2,3)

(5,2) → (1,2)

These new points are exactly the same as the vertices of the initial square.

Option C:

1st transformation (x+3, y+3) will map 4 vertices of the square into points of coordinates such as

(2,1) → (5,4)

(1,2) → (4,5)

(2,3) → (5,6)

(3,2) → (6,5)

next transformation which is the reflection over y=-x+7 gives the coordinates such as

(5,4) → (3,2)

(4,5) → (2,3)

(5,6) → (1,2)

(6,5) → (2,1)

These new points are exactly the same as the vertices of the initial square.

Option D:

1st transformation (x-3, y-3) will map 4 vertices of the square into points of coordinates such as

(2,1) → (-1,-2)

(1,2) → (-2,-1)

(2,3) → (-1,0)

(3,2) → (0,-1)

next transformation which is the reflection over y=-x+2 gives the coordinates such as

(-1,-2) → (4,3)

(-2,-1) → (3,4)

(-1,0) → (2,3)

(0,-1) → (3,2)

These new points are not matching with the vertices of the initial square.

Therefore the correct option is Option D: O(x-3, y-3), reflection over y = -x +2 will not map figure H onto itself.

Learn more about transformation

here: https://brainly.com/question/24632899

#SPJ10

What is the value of x?

Enter your answer in the box.

x =

Answers

Answer:

4

Step-by-step explanation:

In triangle JKL, the side opposite the 30 degree angle, JL, is half the length of the hypotenuse, KL. Thus, JL has a length of

[tex]4 \sqrt{2} [/tex]

In triangle JML, the hypotenuse is

[tex] \sqrt{2} [/tex]

times the length of the leg, meaning that x = 4

what is the equation?
3 + 5(2 + 7n) = 258

Answers

Answer:

n = 7

Explanation:

[tex]\rightarrow \sf 3 + 5(2 + 7n) = 258[/tex]

apply distributive method: a(b + c) = ab + ac

[tex]\rightarrow \sf 3 + 10 + 35n = 258[/tex]

simply add the numbers

[tex]\rightarrow \sf 13 + 35n = 258[/tex]

subtract both sides by 13

[tex]\rightarrow \sf 13-13 + 35n = 258-13[/tex]

simplify the following

[tex]\rightarrow \sf 35n = 245[/tex]

divide both sides by 35

[tex]\rightarrow \sf n = 7[/tex]

Answer:

n = 7

Step-by-step explanation:

1) Subtract 3 from both sides.

5(2 + 7n) = 258 - 3

2) Simplify 258 - 3 to 255.

5(2 + 7n) = 255

3) Divide both sides by 5.

2 + 7n = 255/5

4) Simplify 255/5 to 51.

2 + 7n = 51

5) Subtract 2 from both sides.

7n = 51 - 2

6) Simplify 51 - 2 to 49.

7n = 49

7) Divide both sides by 7.

n = 49/7

8) Simplify 49/7 yo 7.

n = 7

Tried every app none have the answer please help

Answers

Answer:

opotion has not correct answer

answer is 15.

Answer:

[tex]3^\frac{7}{10}[/tex]    or      [tex]\sqrt[10]{3^7}[/tex]

Step-by-step explanation:

[tex]\left(\sqrt{3}\right)\left(\sqrt[5]{3}\right)=\sqrt[10]{3^7}\quad[/tex]

(√3)([tex]\sqrt[5]{3}[/tex]) = √3 · [tex]\sqrt[5]{3}[/tex]

{√3 = [tex]3^{\frac{1}{2}}[/tex]}                 {radical rule: [tex]\sqrt{x}=x^1^/^2[/tex]}

[tex]\sqrt3[/tex][tex]\sqrt[5]{3}[/tex] =  [tex]3^{\frac{1}{2}}[/tex] · [tex]\sqrt[5]{3}[/tex]

{[tex]\sqrt[5]{3}[/tex] = [tex]3^{\frac{1}{5}}[/tex]}                  {radical rule: [tex]\sqrt[n]{x} = x^1^/^n[/tex]}

 [tex]3^{\frac{1}{2}}[/tex] · [tex]\sqrt[5]{3}[/tex] =   [tex]3^{\frac{1}{2}}[/tex] · [tex]3^{\frac{1}{5}}[/tex]     {exponent rule:  [tex]a^x*a^y=a^x^+^y[/tex]}

                (1/2 + 1/5 = 5/10 + 2/10 = 7/10)

              [tex]=3^\frac{7}{10}[/tex]        {opposite of radical rule: [tex]\sqrt[n]{x} = x^1^/^n[/tex] ;  [tex]x^\frac{a}{b}=\sqrt[b]{x^a}[/tex]}

             = [tex]\sqrt[10]{3^7}[/tex]

so, the simplified version of this equation can either be written as:

[tex]3^\frac{7}{10}[/tex]    or      [tex]\sqrt[10]{3^7}[/tex]

hope this helps!!

(I can't clearly see the last option, but if it's either of these, then it's correct)

PLEASE HELP. DON’T NEED DEEP EXPLANATION JUST ANSWERS.

Which of the following are solutions to the equations below?

x^2 + 6x + 9= 20


Check all that apply.

Answers

Answer:

[tex]\large {\textsf{B and E}}\ \implies x_1=-2\sqrt{5}-3,\ x_2=+2\sqrt{5}-3[/tex]

Step-by-step explanation:

Quadratic Formula: [tex]x=\dfrac{-b\pm\sqrt{b^2-4ac}}{2a}[/tex]

Quadratic Equation: ax² + bx + x = 0, where a ≠ 0

Given equation: x² + 6x + 9 = 20

Step 1: Subtract 20 from both sides.

x² + 6x + 9 - 20 = 20 - 20

⇒ x² + 6x - 11 = 0

Step 2: Identify the values of a, b, and c and substitute them in the formula.

⇒ a = 1, b = 6, c = -11

[tex]x=\dfrac{-6\pm\sqrt{\bold{6^2}-4(1)(-11)}}{\bold{2(1)}}\\\\x=\dfrac{-6\pm\sqrt{36\bold{\ - \ 4(-11)}}}{2}\\\\x=\dfrac{-6\pm\sqrt{\bold{36+44}}}{2}\\\\x=\dfrac{-6\pm\sqrt{80}}{2}\\\\x=\dfrac{-6\pm\sqrt{16\times5}}{2}\implies \dfrac{-6\pm\sqrt{\bold{16}}\times\sqrt{5}}{2} \\\\x=\dfrac{-6\pm4\sqrt{5}}{2}[/tex]

Step 3: Separate into possible cases.

[tex]x_1=\dfrac{-6-4\sqrt{5}}{2}\implies \dfrac{-6}{2}+\dfrac{-4\sqrt{5}}{2}\implies \boxed{-3-2\sqrt{5}\ \ \textsf{or}\ -2\sqrt{5}-3}\\\\x_2=\dfrac{-6+4\sqrt{5}}{2}\implies \dfrac{-6}{2}+\dfrac{4\sqrt{5}}{2}\implies \boxed{-3+2\sqrt{5}\ \ \textsf{or}\ \ 2\sqrt{5}-3}[/tex]

Learn more about quadratic equations here:

brainly.com/question/27988045

brainly.com/question/27031173

9 to the power of - half is equal to (as a fraction) 27 to the power of a quarter over 3 to the power of x-1
whats x?
9^(-1/2) = ((27^(1/4))/(3^(x-1)))

Answers

Answer:

[tex]x=\dfrac{11}{4}[/tex]

Step-by-step explanation:

Given:

[tex]9^{-\frac{1}{2}}=\dfrac{27^{\frac{1}{4}}}{3^{(x-1)}}[/tex]

Rewrite 9 as 3² and 27 as 3³:

[tex]\implies (3^2)^{-\frac{1}{2}}=\dfrac{(3^3)^{\frac{1}{4}}}{3^{(x-1)}}[/tex]

[tex]\textsf{Apply exponent rule} \quad (a^b)^c=a^{bc}:[/tex]

[tex]\implies 3^{-1}=\dfrac{3^{\frac{3}{4}}}{3^{(x-1)}}[/tex]

Multiply both sides by [tex]3^{(x-1)}[/tex] :

[tex]\implies 3^{-1} \cdot 3^{(x-1)}=3^{\frac{3}{4}}[/tex]

[tex]\textsf{Apply exponent rule} \quad a^b \cdot a^c=a^{b+c}:[/tex]

[tex]\implies 3^{(-1+x-1)}=3^{\frac{3}{4}}[/tex]

[tex]\implies 3^{(x-2)}=3^{\frac{3}{4}}[/tex]

[tex]\textsf{Apply exponent rule} \quad a^{f(x)}=a^{g(x)} \implies f(x)=g(x):[/tex]

[tex]\implies x-2=\dfrac{3}{4}[/tex]

[tex]\implies 4(x-2)=3[/tex]

[tex]\implies 4x-8=3[/tex]

[tex]\implies 4x=11[/tex]

[tex]\implies x=\dfrac{11}{4}[/tex]

there were 3 parts to Ritas race she ran the first part which was 4/9 of the total distance, in 20 minutes she ran the second part which was 2/5 of the remaining distance, in 12 minutes. She finally ran the third part in 15 minutes at the speed of 300 meters per minute.
How long was the first part of the race? What was Ritas speed in that section of the race?

Answers

Answer:

3/5 ..................................  6/9

Step-by-s4tep explanation:

Answer:

300 meters per minute

Step-by-step explanation:

Step 1

Determine the fraction of the total distance for each part of the race.

Given:

1st part of race = 4/9 of total distance2nd part of race = 2/5 of remaining distance

[tex]\begin{aligned} \implies \sf 2nd\:part & = \sf \dfrac{2}{5}\:of\:remaining\:distance\\ & = \sf \dfrac{2}{5} \times\left(1-\dfrac{4}{9}\right)\\& = \sf \dfrac{2}{9}\:of\:total\:distance\end{aligned}[/tex]

[tex]\begin{aligned}\implies \sf 3rd\:part & = \sf 1-1st\:part-2nd\:part\\& = \sf 1-\dfrac{4}{9}-\dfrac{2}{9}\\& = \sf \dfrac{1}{3}\:of\:total\:distance\end{aligned}[/tex]

Step 2

Determine the distance of the third part of the race.

Given:

time = 15 minutesspeed = 300 meters per minute

[tex]\begin{aligned}\sf Distance & = \sf speed \times time\\& = \sf 300 \times 15\\& = \sf 4500 \:\:meters\end{aligned}[/tex]

Step 3

If the third part of the race (which is 1/3 of the total distance) is 4500m, then the distance of the whole race is:

[tex]\begin{aligned} \sf Total\:distance & = \sf 4500 \times 3\\& = \sf 13500\:meters \end{aligned}[/tex]

Step 4

Determine the distance of the 1st part of the race:

[tex]\begin{aligned}\sf Distance\:of\:1st\:part\:of\:race & = \sf \dfrac{4}{9} \:of\:total\:distance\\& = \sf \dfrac{4}{9} \times 13500\\& = \sf 6000\:meters \end{aligned}[/tex]

Step 5

Determine the speed of the 1st part of the race:

Given:

time = 20 minutesdistance = 6000 m

[tex]\begin{aligned}\sf Speed\:of\:1st\:part\:of\:race & = \sf \dfrac{distance}{time}\\& = \sf \dfrac{6000}{20}\\ & = \sf 300\:meters\:per\:minute\end{aligned}[/tex]

Hi everyone! I would be really glad if anyone could help me solve these step by step, I tried solving it on my own and viewing lessons, but I still don't understand. Thank you.

Answers

The equation of the function is f(x) = x(x + 2)(x -1)(x -3)

The function end behavior

From the graph, we have the following highlight:

As x increases, the function values increaseAs x decreases, the function values increase

The above means that:

f(x) approaches positive infinity irrespective of the x value

Hence, the end behavior of the function is:

[tex]\mathrm{as}\:x\to \:+\infty \:,\:f\left(x\right)\to \:+\infty \:,\:\:\mathrm{and\:as}\:x\to \:-\infty \:,\:f\left(x\right)\to \:+\infty \:[/tex]

The sign of the leading coefficient

Since, the function opens upward.

Then the sign of the leading coefficient is positive

The zeros of the function

This is the point, where the graph crosses the x-axis.

From the graph, we have the zeros to be:

x = -2, x = 0, x = 1 and x = 3

Since the graph crosses the x-axis at this point, then the multiplicity of the zeros is 1

The equation of the function

In (c), we have:

x = -2, x = 0, x = 1 and x = 3

Rewrite as:

x + 2= 0, x = 0, x - 1 = 0 and x - 3 = 0

Multiply these values

f(x) = x(x + 2)(x -1)(x -3)

Hence, the equation of the function is f(x) = x(x + 2)(x -1)(x -3)

Read more about polynomials at:

https://brainly.com/question/4142886

#SPJ1

[tex]\sf\large\green{\underbrace{\red{Befikra*}}}:[/tex].

The first term of an arithmetic sequence is 1 and the sum of the first four terms is 100. Find the first four terms​

Answers

First term f = 1

If first four terms are f, f + d, f + 2d, f + 3d

f + f + d + f + 2d + f + 3d = 100

4f + 6d = 100 (divided by 2 , both sides )

2f + 3d = 50

2 + 3d = 50

3d = 50 – 2

3d = 48

d = 48/3

d = 16

The arithmetic sequence is 1, 17, 33, 49, ………….

Answer:

[tex]\sf\large\blue{\underbrace{\red{itz \: jass*}}}:[/tex]

Step-by-step explanation:

[tex]\sf\large\green{\underbrace{\red{hlo \: \: sat \: shri \: akal \: ji \: \: \: *}}}:[/tex]

Livia eats a chicken drumstick with 11 grams of protein. She also eats x cheese sticks, each with 7 grams of protein. The table shows y , the total number of grams of protein that Livia will consume if she eats x cheese sticks. Livia may eat only part of a cheese stick, so x may not always be a whole number.
What is the range of the function?

Answers

Livia eats a chicken drumstick with 11 grams of protein. She also eats x cheese sticks, each with 7 grams of protein. The table shows y , the total number of grams of protein that Livia will consume if she eats x cheese sticks. Livia may eat only part of a cheese stick, so x may not always be a whole number.

What is the range of the function?

Which ordered pair would form a proportional relationship with the point graphed below?

Answers

Answer:

  (c)  (-30, 10)

Step-by-step explanation:

Two points will be on the same graph of a proportional relationship if the ratio of values y/x is the same.

__

The given point (x, y) = (60, -20) has a ratio k = y/x = -20/60 = -1/3.

The offered points have y/x ratios ...

(a) 30/-10 = -3

(b) -15/30 = -1/2

(c) 10/-30 = -1/3 . . . . . . the point you are looking for is (-30, 10)

(d) -30/80 = -3/8

question attached below

Answers

If the first inequality is written as:

3x ≤ -12

Then the correct option is D.

If it is written as it is in the problem, then neither of the graphs is the correct option.

How to solve the compound inequality?

Here we have the inequalities:

3x ≤ 12

2x + 3 > 11

(notice that in all the options, the first inequality has a negative sign, while in the two given inequalities there are no negative sign. This means that you will never get the correct answer if you use the given information, as the problem is incorrectly written).

First, we need to isolate x on both inequalities, we will get:

3x ≤ 12

x ≤ 12/3

x ≤ 4

(As you can see, because we don't have the negative sign, this inequality is different to all the ones in the options).

And for the other:

2x + 3 > 11

2x > 11 - 3

x > 8/2

x > 4

So neither of the options is actually correct.

If we instead write the first inequality as:

3x ≤ -12

We would solve this as:

x ≤ -4

And in that case, our two inequalities are:

x ≤ -4

x > 4

So the correct graph is graph D.

If you want to learn more about inequalities:

https://brainly.com/question/18881247

#SPJ1

please help mee, (30 points will give brainliest!!!)

Answers

Answer:

-4

+3

12

-0.5, 12.25

x = -0.5

Step-by-step explanation:

The x intercepts are the values of x when y = 0 ie the roots of the equation
[tex]-x^2 -x +12 = 0[/tex]

or

[tex]x^2 + x - 12 = 0[/tex]

We can re-write the above as:
(x+4) (x-3) = 0
This gives the two roots as x = -4 and x = +3. Leftmost (smallest) root is -4 and rightmost(largest) root is +3

y intercept is when x = 0. Plugging into the original equation, y value at x = 0 is 12

Vertex x value is given by the formula -b/2a where a is the coefficient of x^2 and b the coefficient of x

Here a = -1, b= -1 so vertex x value = - (-1)/(-1).2 = - 1/2 = -0.5

Plugging this value of x into the original function gives the vertex y value
[tex]y = - (0.5^{2}) - (-0.5) + 12\\= -0.25 + 0.5 + 12 = 12.25[/tex]

The line of symmetry is the vertical line corresponding to the vertex x value so line of symmetry is at x = -0.5

The graph of the quadratic function shows these values

8.
(05.04 LC)

The amount of money in tips earned by four restaurant servers waiting on 10 tables is represented by the following data sets.

Alyssa {3, 6, 2, 8, 12, 14, 5, 7, 7, 8}
Bryant {9, 2, 7, 50, 0, 5, 2, 8, 6, 8}
Camila {1, 9, 10, 3, 0, 12, 10, 9, 8, 2}
Devon {4, 2, 8, 15, 20, 7, 5, 0, 6, 2}

Which data set has the greatest interquartile range? (1 point)
Alyssa

Bryant

Camila

Devon

Answers

Camila's data set has the greatest interquartile range and the value of IQR = 8 option third is correct.

What is the range?

It is defined as the difference between the maximum value in the data set to the minimum value in the data set.

We have:

The amount of money in tips earned by four restaurant servers waiting on 10 tables is represented by the following data sets:

Alyssa {3, 6, 2, 8, 12, 14, 5, 7, 7, 8}

Bryant {9, 2, 7, 50, 0, 5, 2, 8, 6, 8}

Camila {1, 9, 10, 3, 0, 12, 10, 9, 8, 2}

Devon {4, 2, 8, 15, 20, 7, 5, 0, 6, 2}

The IQR for Alyssa:

IQR = Q3 - Q1

IQR = 8 - 5 = 3

The IQR for Bryant:

IQR = Q3 - Q1

IQR = 8 - 2 = 6

The IQR for Camila:

IQR = Q3 - Q1

IQR = 10 - 2 = 8

The IQR for Devon:

IQR = Q3 - Q1

IQR = 8 - 2 = 6

Thus, Camila's data set has the greatest interquartile range and the value of IQR = 8 option third is correct.

Learn more about the range here:

https://brainly.com/question/17553524

#SPJ1

Work out m and c for the line:

y = 2 − 3 x

Answers

Answer:

m = -3

c = 2

Step-by-step explanation:

This equation is given in slope-intercept form. The general structure of these equations is:

y = mx + c

In this form, "m" represents the slope and "c" represents the y-intercept. As you have been given the entire equation, the only thing you have to do is rearrange the equation to find which values correspond with the variables.

y = 2 - 3x  -----> y = -3x + 2

Therefore,

m = -3

c = 2

Y = mx + c
This is the equation to find linear graphs
You have
Y = 2 - 3x
Rearranged
Y = + 2 - 3x
Y = -3x + 2
So

What do you think m and c are if
Y = mx + c and

Y = -3x + 2

Helppp!!!
[tex] \dfrac{2}{15} \div \dfrac{20}{3} [/tex]
.........​

Answers

[tex]{\huge \underline{{ \fbox \color{red}{A}}{\fbox \color{green}{n}}{\fbox \color{purple}{s}}{\fbox \color{brown}{w}}{\fbox \color{yellow}{e}}{\fbox \color{gray}{r } }}}[/tex]

[tex] \sf \dfrac{2}{15} \div \dfrac{3}{20} = \dfrac{2 \times 20}{15 \times 3} = \dfrac{40}{45} = \dfrac{8}{9} [/tex]

Answer :-

[tex] \: \: \: \: \: \: \: \: \: \: \: \: \: \: \: \: \: \: \: \: \: \: \: \: \: \: \: \sf \Rrightarrow \dfrac{8}{9} [/tex]

I hope I've helped : )

Answer:

[tex]\dfrac{1}{50}[/tex]

Step-by-step explanation:1. We multiply.

[tex] \cfrac{2}{15} \times \cfrac{3}{20} [/tex]

2. We join the fractions and put it only in 1.

[tex] \cfrac{2 \times 3}{15 \times 20} [/tex]

3. We multiply the one above which is 2 × 3.

[tex] \cfrac{6}{15 \times 20} [/tex]

4. Now we will multiply the below.

[tex] \cfrac{6}{300} [/tex]

5. Now we simplify.

[tex] \cfrac{1}{50} [/tex]

-------------------------------------------------------------------------------------------------------

Learn more about solving fractions here:

https://brainly.com/question/16627508

https://brainly.com/question/29121733

Helpppp I stuck so please help
Me

Answers

Answer = #2 - 1/3^3

Step-by-step explanation:

if y= 2f(g(x)), then d^2y/dx^2=​

Answers

Use the chain rule to differentiate both sides of the given equation with respect to [tex]x[/tex] :

[tex]y = 2 f(g(x))[/tex]

[tex]\dfrac{dy}{dx} = y' = 2 f'(g(x)) g'(x)[/tex]

Differentiate again with the product and chain rules to get the second derivative :

[tex]\dfrac{d^2y}{dx^2} = y'' = \boxed{2 f'(g(x)) g''(x) + 2 f''(g(x)) g'(x)^2}[/tex]

(D)

Without using the formula, work out the gradient of the graph shown.
Give your answer in its simplest form?

Answers

Answer:

Slope is 3/4

Step-by-step explanation:

Gradient means slope. So, to find the slope without using the formula, look at two points then count up until you're in line with the other point, then count right until you reach the other point. Remember it must be written as rise/run.

I hope this explanation makes sense!

The points I will use is (0,-2) and (4,1).

Slope is 3/4.

Hope this helps!

If not, I am sorry.

jackie brought 9 plants ​

Answers

Answer:

t+s = 9

4t+9s=51

tulips and succulents combined is 9 plants

the amount of money they have combined is $51

Can someone help me please?

Answers

Answer:

C.

Step-by-step explanation:

The reds are counting up in whole numbers by one integer.

Hope this helps!

If not, I am sorry.

The exterior of a solid cone is painted. The height of the cone is 11.4 centimeters, and the diameter of its opening is 5 centimeters.

What is the surface area of the solid cone requiring paint to the nearest square centimeter?

The surface area of the solid cone requiring paint rounded to the nearest whole number is ___________ square centimeters.

Answers

Answer:

111 cm^2

Step-by-step explanation:

h = 11.4

d = 5  so  r (radius) = 2.5

Surface Area of a Cone Formula: πr^2 + πrs

s represents the slant height of the cone, lets represent this with a drawing.

We have the values of h and r, but we don't have the value of s, meaning we have to find it. We can do this by using pythagorean theorem.

a^2 + b^2 = c^2

These three values can come together to create a right triangle which we can seperate from the rest of the diagram, now we can find s.

s = √(11.4^2 + 2.5^2)   so  s = 11.67

now that we have all of the pieces, we can plug back into the original surface area formula. πr^2 + πrs

π(2.5^2) + π(2.5)(11.67) = 111.29 cm^2 or 111 cm^2

The surface area of the solid cone requiring paint will be 111 square cm.

What is the surface area of a cone?

Let d be the diameter of the base circle, l be the slant height, and h be the height of the cone.

Then the lateral surface area of the cone will be

SA = πrl + πr²

Then the slant height is given as

l² = r² + h²

The exterior of a solid cone is painted. The height of the cone is 11.4 centimeters, and the diameter of its opening is 5 centimeters.

Then the radius of the base circle will be

r = 5 / 2

r = 2.5 cm

The length of the slant height will be

l² = 11.4² + 2.5²

l² = 136.21

 l = 11.67

Then the surface area of the solid cone requiring paint will be

SA = π × 2.5 × 11.67 + π × 2.5²

SA = 29.18π + 6.25π

SA = 35.43π

SA = 111.3

SA ≈ 111 square cm

The surface area of the solid cone requiring paint will be 111 square cm.

More about the surface area of a cone link is given below.

https://brainly.com/question/23877107

#SPJ5

What is the approximate length of the radius, r? use 3.14 for . round to the nearest inch. 12 inches 24 inches 38 inches 46 inches

Answers

Answer:

2×3.14×r=75

6.28 r=75

r=75/6.28=11.943=12 inch (nearly)

If with 150 grams of ground meat a tortilla is prepared to make a hamburger. How many tortillas can I make with one and a half kilograms of ground meat?​

Answers

Answer:

You can make 10 tortillas with one and a half kilograms of ground meat

Step-by-step explanation:

one and a half kilograms to grams

1.5kg x 1000 = 1500g

150 grams prepared to make 1 hamburger

than, 150 x h = 1500

h = 1500 / 150

h = 10

Answer:

10 tortillas

Step-by-step explanation:

First convert kilograms to grams:

   1 kg = 1000 g

⇒ 1.5 kg = 1500 g

Let x = number of tortillas made with 1500 g of meat

Set up a ratio of tortillas and meat and solve for x:

⇒ 1 tortilla : x tortillas = 150 g meat : 1500 g meat

[tex]\sf \implies \dfrac{1}{x}=\dfrac{150}{1500}[/tex]

[tex]\sf \implies 1 \cdot 1500= 150 \cdot x[/tex]

[tex]\sf \implies 1500=150x[/tex]

[tex]\implies \sf x=\dfrac{1500}{150}[/tex]

[tex]\implies \sf x=10[/tex]

Therefore, 10 tortillas can be made with 1.5 kg of ground meat.

how many feet? help me please​

Answers

Answer:

[tex]\fbox {400.41 feet}[/tex]

Step-by-step explanation:

Here we need to take the tan ratio of the angle.

⇒ tan 4° = 28/x = 0.0699268119

⇒ x = 28/0.0699268119

⇒ x = 400.41 feet

40 POINTS PLS HELP PLS PLS PLS PLS PLS PLS PLS PLS PLS PLS /J IM LAZY NO PLS HELP

Answers

Answer:

y = -0.5x - 3.5

Explanation:

Take two points: (1, -4), (3, -5)

Find slope:

[tex]\sf slope: \dfrac{y_2 - y_1}{x_2- x_1} \ \ where \ (x_1 , \ y_1), ( x_2 , \ y_2) \ are \ points[/tex]

[tex]\rightarrow \sf slope \ (m) : \dfrac{-5-(-4)}{3-1} = -\dfrac{1}{2}[/tex]

Equation:

[tex]\sf y- y_1 = m (x - x_1)[/tex]

[tex]\rightarrow \sf y- (-4) = -\dfrac{1}{2} (x -1)[/tex]

[tex]\rightarrow \sf y+ 4 = -\dfrac{1}{2} x + \dfrac{1}{2}[/tex]

[tex]\rightarrow \sf y = -\dfrac{1}{2} x + \dfrac{1}{2} - 4[/tex]

[tex]\rightarrow \sf y = -0.5 x -3.5[/tex]

The formula of a line is:

[tex]f(x) = kx + b[/tex]

k — stands for tg of line angle
b — stands for the y value of a point where the line crosses y axis.


We can see 2 points:
A (1, -4)
B (3, -5)

We can make a system of equations:

[tex]k*1+b=-4\\k*3+b=-5\\[/tex]

From second equation:

[tex]b = -5 - 3k[/tex]

Let's put b in first equation:

[tex]k-5-3k=-4\\-2k=1\\k=- \frac{1}{2}[/tex]

Now we can find b:

[tex]b = -5 -3*- \frac{1}{2}\\b = -5 + 1.5\\b = -3.5[/tex]

The answer is:
-0.5x -3.5

Other Questions
Federal regulations require Medicaid to establish and maintain a(n) __________ program, which safeguards against unnecessary or inappropriate use of Medicaid services or excess payments and assesses the quality of those services. a. recipient eligibility verification b. early and periodic screening, diagnostic, and treatment c. eligibility verification d. surveillance and utilization review Are any factors in the equation of photosynthesis more critical for plants that live on land than for plants that live underwater? PLEASE HELP ASAP!!!!!!!1Consider the series 1/4+3/2+11/4+4+21/4+....Does the series converge or diverge?Select answers from the drop-down menus to correctly complete the statements.The series _____ (diverges or converges). You Consider conclude this because the series is _________ (Arithmetic, Geometric and the absolute value of the common ratio is greater than 1, Geometric and the absolute value of the common ratio is less than 1, neither arithmetic nor geometric) If f(x)=7^x and g(x)=15x/x+9, find a simplified formula for: g(f(x))= A rectangular prism has a length of 8 in., a width of 4 in., and a height of 2 1/4 in.The prism is filled with cubes that have edge lengths of 14 in.How many cubes are needed to fill the rectangular prism?Enter your answer in the box.To fill the rectangular prism, cubes are needed. What is the safest way to change direction while driving?. Hi, could you help me with this substitution question?I'd really appreciate if you added workings out as I'm terribly confused.Thanks, All definitions are correct except: Leverage: using other people's money. Equity buildup: As the loan is paid, the amount an investor has invested increases, thus decreasing the loan and increasing equity. Liquidity: how fast the property can be sold. Basis: an income tax term meaning how much the property is currently worth. What is wrong with this page layout? A. The image should be placed closer to the text.B. It isn't clear what the image is showing, or how it's relevant.C. The image should be placed at the bottom of thepage.D. There should be more white space between the image and thetext. Read this passage:High school is just a couple years away. I am going to make my high school years unforgettable. I'm going to choose lots of exciting elective courses so my school day is always interesting. I'm going to join several clubs so I can meet new friends who like the same things that I do. I'm also going to have fun by attending football games, dances, and as many other social events as possible. With this plan, I'm guaranteed to have a great timeas long as I keep my grades up and stay out of trouble.The main idea of this paragraph is written in bold. Writing in complete sentences, list three details the author uses to support it. What do plants need to survive and reproduce Which buffer can be suitable to prepare a solution that needs to be buffered at pH 4? Explain yourchoice.A. Formic acid/Formiate bufferB. Carbonic acid/Bicarbonate bufferC. Ammonia/Ammonium bufferD. Any buffer will work.E. None of the buffers will be suitable.34 Which of the following will form a buffer when mixed together? Explain Aude slipped on ice going 7.6 m/s .What was her velocity before she hit the wal? Describe the trading of enslaved people that had existed in africa before the arrival of europeans. explain what caused the slave trade to expand. heyyplease help me with these answerstheyre quite simple but i dont really know how to explain the answers1. Tulips are pollinated by insects. explain three ways the flower is adapted for this type of pollination2. explain the differences in the types of pollen produced by insect-pollinated amd wind-pollinated plants.thankss Which is an example of an income deduction? Select the correct text in the passage. Which sentence states the author's thesis? excerpt from Presidential Retreat adapted from the National Parks Service President Franklin D. Roosevelt was accustomed to seeking relief from hot Washington, D.C. summers and relaxing on weekends aboard the presidential yacht Potomac or at Hyde Park, NY. In 1942, the U.S. Secret Service were very concerned about the President's continued use of the Potomac. World War II had brought an attack on Pearl Harbor and had drawn German U-boats close in Atlantic waters. Presidential safety and Presidential health were concerns. The muggy climate of Washington, D.C. was considered detrimental to his health, affecting his sinuses. A new retreat, a place to relax, within a 100 mile radius of Washington, D.C. and in the cool mountain air was sought. Several sites were considered, but Camp HI- Catoctin in the Catoctin Recreational Demonstration Area was selected after the President's first visit on April 22, 1942. A camp was already built on the site and the estimated conversion cost was $18,650. It was also almost 10 degrees cooler than in Washington, D.C. The camp for federal employees' families became the camp of one federal employee, the President of the United States. Roosevelt quickly renamed the camp "Shangri-La" from James Hilton's 1933 novel, Lost Horizon. What is the surface area of a sphere with radius 3? Campbell, a single taxpayer earns $400,000 in taxable income and $2,000 in interest from an investment in State of NY bonds. How much federal tax will she owe? What is her average tax rate? What is her effective tax rate? What is her current marginal tax rate? The probability distribution for a random variable x is given in the table. Probability Find the probability that x=-10